A rectangular prism has a length of 19 meters, a height of 13 meters, and a width of 14 meters. What is its volume, in cubic meters?

Answers

Answer 1

Answer:

V=whl=14·13·19=3458m³

Answer 2

Answer:

1152.66 m³

Step-by-step explanation:

Volume = ⅓ × base area × height

= ⅓ × (length × width) × height

= ⅓ × 19 × 14 × 13

= ⅓ × 3458

= 1152.66 m³


Related Questions

need help with this one if someone doesnt mind helping. thank you!

Answers

Answer:

wdymm?

Step-by-step explanation:

WILL GIVE BRAINLIEST!!! SELECT MULTIPLE ANSWERS!!!

Answers

Step-by-step explanation:

soory i can't give an answer

I need help with these soon for the marking period

Answers

Answer:

1) x = 62.2

2) x = 35.9

3) x = 61.9

4) x = 53.1

simplifey the expression 24x + 32 + 4x +3 what is the coefficient of x

Answers

28 is the coefficient and the simplified expression is 28x + 35

radius of a semi circle

Answers

The area for semi circle is: (pi)(r^2)/2

A= 3.142 x (8.2)^2 / 2

A= 110.8 cm^2
A=3.142*r^2
Area of the circle= 3.142*8.2^2
=211.26
Because it’s a semicircle, divide by two.
211.26/2 = 105.6

A frozen yogurt shop offers frozen yogurt in the sizes shown.

The cost per cubic inch is $0.10 for each container’s contents. What is the difference between the costs of yogurt in the two containers if each is filled with yogurt? Round to the nearest cent.

Cost of the yogurt in cylinder: $
____

Cost of the yogurt in cone:$
____
Difference in cost:$
____

Answers

The cost for the cylindrical container will be $2.8 and for the conical container, $0.94 and the difference is $1.86

What is volume?

Volume is defined as the space occupied by any object in the three-Dimensions. All three parameters are required for the volume like radius length in the case of cylinders and cones.

Given that:-

The cost per cubic inch is $0.10 for each container’s contents

The volume of a cylinder:-

V   =   πr²h

V   =   π  x  (1.5)²  x  4  =  28.27 cubic inches

So the cost will be = 28.27  x   0.10  =   $2.8

The volume of the cone:-

V   =  (1  /  3  ) πr²h

V   =  (1  /  3  )   π  x  ( 1.5 )²  x  4 =  9.42 cubic inches

The cost will be =  9.42   x   0.10  = $0.94

The difference between the two costs will be:-

=    $ 2.8   -    $0.94

=   $1.86

Therefore the cost for the cylindrical container will be $2.8 and for the conical container, $0.94 and the difference is $1.86

To know more about Volume follow

https://brainly.com/question/1972490

#SPJ1

Answer: cost for the yogurt cylinder 2.83 cost of the yogurt  cone 0.94 different cost 1.89

Step-by-step explanation:

use formula to find the volume

then multply both 0.10

round to the nearest hundredth

then subtract

What is the circumference of the following circle? Diameter=4 and Raidar=2

Answers

Answer:

Step-by-step explanation: The diameter is 4 cm which means the radius is 2 cm. Diameter divided by two will give you the radius. The formulae for circumference is 2 π⋅r. So the circumference would be 4π

From his eye, which stands 1.64 meters above the ground, Keshawn measures the angle of elevation to the top of a prominent skyscraper to be 33 degrees. If he is standing at a horizontal distance of 385 meters from the base of the skyscraper, what is the height of the skyscraper? Round your answer to the nearest tenth of a meter if necessary.

Answers

The height of the scyscraper will be equal to H=251.66 meters

What is trigonometry?

The branch of mathematics set up a relatioship between the angle and the sides of the right angle triangle is called as trigonometry.

Here we have the following data:-

The height of the observer's eye=1.63 meters

The angle which eye is making withe the building=33

The horizontal distance = 385 meters

So the height of the building is calculated as:-

[tex]Tan\theta=\dfrac{P}{B}\\\\\\tan\theta=\dfrac{H-1.64}{385}\\\\ \\\\tan33=\dfrac{H-1.64}{385}\\\\\\\\[/tex]

H=251.66 meters

hence height of the scyscraper will be equal to [tex]H=251.66\ meters[/tex]

To know more about Trigonometry follow

https://brainly.com/question/24349828

#SPJ1

Answer:251.7

Step-by-step explanation:

Which expression is equal to

Answers

The expression that is equal to that is two to the eighth which in the second choice. 2^8.

braden scored 9 fewer points on his math test than ava did if ava scored an 86 what was bradens score

Answers

Subtract 9 from 86


86 - 9 = 77

Answer : Braden’s score was 77

Describe the graph that is produced by the equation (x - 7)2 + (y + 5)2 > 25.
The conic represented by the equation is a(n)
The solution set is
v include the boundary.

Answers

The conic section which is represented by the equation is a circle. The solution set of this equation are (7.0),(2.-5),(7.-10),(12.-5).

What is the equation of circle?

The equation of the circle is the equation which is used to represent the circle in the algebraic equation form, with the value of center point in the coordinate plane and measure of radius.

The standard form of the equation of the circle can be given as,

[tex](x-h)^2+(y-k)^2=r^2[/tex]

Here (h,k) is the center of the circle and (r) is the radius of the circle.

The given inequality equation is,  

[tex](x - 7)^2 + (y + 5)^2 > 25\\(x - 7)^2 + (y + 5)^2 > 5^2[/tex]

This equation is similar to the equation of circle. Thus, the conic represented by the equation is a circle. The center and radius of this circle is,

[tex](h,k)=(7,-5)\\r=5[/tex]

The solution set of this circle is the all the point which lies on this circle. In the attached graph, the solution set is shown which are (7.0),(2.-5),(7.-10),(12.-5).

Thus, the conic section which is represented by the equation is a circle. The solution set of this equation are (7.0),(2.-5),(7.-10),(12.-5).

Learn more about the equation of circle here;

https://brainly.com/question/1506955

#SPJ1

Answer:

A circle with a center (7,–5) and d unbounded and does not

Step-by-step explanation:

The conic represented by the equation is a(n)

✔ circle with a center (7,–5)

.

The solution set is

✔ unbounded and does not

include the boundary.

I currently have a 32% in Math, but assuming I got a 95 on the test we took, what would be my grade?

Answers

Answer:

Congratulations you have an A on your test!!!!

Step-by-step explanation: The grade scale :)

A+ 97-100 4.0

A 93-96 4.0

A- 90-92 3.7

B+ 87-89 3.3

B 83-86 3.0

B- 80-82 2.7

C+ 77-79 2.3

C 73-76 2.0

C- 70-72 1.7

D+ 67-69 1.3

D 65-66 1.0

E/F Below 65 0.0

help me asap the pdf is down below

Answers

The mathematics properties and solutions to the pdf are listed below

Q1.i. Multiplicative commutative  property: [tex]\frac{1}{4}[/tex] * [tex]\frac{3}{7}[/tex] = [tex]\frac{3}{7}[/tex] *  [tex]\frac{1}{4}[/tex]  = [tex]\frac{3}{28}[/tex]ii.  Distributive  property : [tex]\frac{2}{3}[/tex] * ( [tex]\frac{5}{7}[/tex] - [tex]\frac{9}{11}[/tex] )iii. Associative property:  [tex]\frac{6}{5}[/tex] + ( [tex]\frac{1}{4}[/tex] + [tex]\frac{8}{9}[/tex] )  =  ( [tex]\frac{6}{5}[/tex] +  [tex]\frac{1}{4}[/tex] ) + [tex]\frac{8}{9}[/tex] Q2i. [tex]\frac{12}{11}[/tex] * [tex]\frac{21}{5}[/tex] +  [tex]\frac{12}{11}[/tex] * [tex]\frac{9}{5}[/tex]  =    [tex]\frac{12}{11}[/tex] * ( [tex]\frac{21}{5}[/tex] + [tex]\frac{9}{5}[/tex] )ii. [tex]\frac{1}{11}[/tex] * [tex]\frac{17}{10}[/tex]  - [tex]\frac{7}{10}[/tex]  *   [tex]\frac{1}{11}[/tex]  =   [tex]\frac{1}{11}[/tex]  * ( [tex]\frac{17}{10}[/tex]  - [tex]\frac{7}{10}[/tex] )Q3.i. [tex]\frac{72}{11}[/tex]  =   [tex]\frac{72}{11}[/tex]ii.  [tex]\frac{13}{11}[/tex]  =    [tex]\frac{13}{11}[/tex]iii. -49 = (-49)iv. [tex]\frac{121}{81}[/tex]  =   [tex]\frac{121}{81}[/tex] v.  [tex]\frac{265}{72}[/tex]  =   [tex]\frac{265}{72}[/tex]vi.  [tex]\frac{265}{72}[/tex]  =   [tex]\frac{265}{72}[/tex]

Meaning of mathematics properties

Mathematics properties is a characteristics or feature that is applied to a mathematics equation or given to a set to a definite result.

Mathematics properties have three major properties which are the commutative property, associative property and the distributive property.

In conclusion, the mathematics properties and solutions to the equation are listed above.

Learn more about mathematics properties: https://brainly.com/question/17210986

#SPJ1

Suppose your car is rated at 38 mi/gal for city driving. If you wanted to write your friend in Spain about your car’s mileage, what ratings in kilometers per liter would you report?

Answers

One gallon is equal to 4.546 liters and one mile is equal to 1.609 kilometers. Then the ratings in kilometers per liter will be 13.45.

What is conversion?

Conversion means to convert the same thing into different units.

Suppose your car is rated at 38 mi/gal for city driving. If you wanted to write your friend in Spain about your car’s mileage.

Then the ratings in kilometers per liter will be

We know one gallon is equal to 4.546 liters and one mile is equal to 1.609 kilometers.

Then we have

[tex]\rm \rightarrow \dfrac{38 \times 1.609 \ km}{4.546 \ liters}[/tex]

On solving further, we have

→ 13.449 ≅ 13.45 km per liter

More about the conversion link is given below.

https://brainly.com/question/9414705

#SPJ1

helpppppppppppppppppppp

Answers

Answer:

see explanation

Step-by-step explanation:

under a reflection in the y- axis

a point (x, y ) → (- x, y ) , then

(- 4, - 7 ) → (4, - 7 )

Question 4 of 10
Which of the following is the equation of a line in slope-intercept form for a
a
line with slope = 2/3 and yintercept at (0, -2)?

Answers

Answer:

A is in line with slope =2/3 and yintercept at (0,-2)

Sadida needed to reorder shirts for her clothing store. She ordered 63 medium shirts and 27 large shirts. What was the ratio of medium to large shirts.

Answers

Step-by-step explanation:

the ratio medium to large was

63/27 = 7/3

What is the scale factor?

Answers

Answer:

This is correct answer if it help you please remember me in your pary thanks

What is the surface area of the cylinder with height 3 mi and radius 8mil? Round your answer to the nearest thousandth

Answers

Answer:

553.143 mi²

Step-by-step explanation:

Surface Area (Cylinder) :

2πr (r + h)2 x 22/7 x 8 (8 + 3)16 x 22/7 x 11176 x 22/73872/7553.143 mi²

Answer:

What is the surface area of the cylinder with height 3 mi and radius 8mil? Round your answer to the nearest thousandth

Step-by-step explanation:

SOLUTION

HEIGHT -3

RADIUS - 8

AREA - 2πr ( r+h)

[tex] 2 \times \frac{22}{7} \times 8(8 + 3) \\ \\16 \times \frac{22}{7} \times 11 \\ \\ 176 \times \frac{22}{7} \\ \\ \frac{3872}{7} \\ = 553.143[/tex]

A 15 oz can of green beans usually costs $0.99. This week, a supermarket chain is advertising 2 cans for $1.70. Is this a bargain? Show your work and explain why or why not it will save money and by how much ?

Answers

Answer:

It is a bargain because 0.99 + 0.99 = 1.98 which is more than 1.70. You would be paying less with this deal. It will save you 28 cents

Step-by-step explanation:

what is 100x3856+284 dived by 6 x384

Answers

Answer:

179 R 1584

Part 1

6 × 384

6 × 4 = 24(carry the 2)

6 × 8 + 2 = 50(carry the 5)
6 × 3 + 5 = 23

Put it all together into one number

23 + 0 + 4 = 2,304

Part 2

3,856 + 284

6 does not add to 4 so it's 0

5 + 8 = 14(carry the 1 and add 1)

8 cannot add onto 2 so use the one you borrowed and carry. = 1

3 + 1 = 4

Put it together

4 + 1 + 4 + 0 = 4,140

Part 3

414,000

Simply add the two 0's in 100 to 4,140

Part 4

414,000 ÷ 2,304

Turn 4,140 into the thousands place temporarily.

4140

-2304

=1,836

Drop 0 to the result

18360

-16128

=2,232

Drop 0 again

22320

-20736

=1,584

Finally, you have a result.

179, Remainder 1584

the range of a set of numbers is 13.25
the smallest number is -3.875
workout the largest number

Answers

Answer:9.375

Step-by-step explanation: mark me the brainly-est

Help me please help me

Answers

Answer:

e= 543

Step-by-step explanation:

Hello!

902-e=359, so 902-359=e.

8  9 12902 359------543

Answer:

E=543

Step-by-step explanation:

To find the variable for E Subtract 902-359 and you get 543 or E

5Look at the picture to the right.
Which relationships have the same constant of proportionality between y and x as in the equation?
Select all that apply

I know from the charts it's C but from the latest tables I don't know.

Answers

*Correct Question:

Which relationships have the same constant of proportionality between y and x as the equation 3y=27x?

Answer:

A, B, C

Step-by-step explanation:

Given that [tex]3y=27x[/tex], we can simplify to get a proportionality statement that exists between X and y. Thus

[tex]3y=27x[/tex]

Divide both sides by 3

[tex]\frac{3y}{3}=\frac{27x}{3}[/tex]

[tex]y=9x[/tex]

Thus, we can say, y would always be 9 times the quantity of x.

From the options given, examine which options have this proportionality statement as well.

Option A, y = 9x is same as the statement.

Option B, 2y = 18x, conforms to the same statement. If we simply further, we would have y = 9x

Option C, the graph shows that when x = 1, y = 9. This also confirms to the same constant of proportionality in the given equation.

Option D and E do not have same constant of proportionality.

The right options are A, B, and C.

Which expression has a value of 80?

Answers

Type each expression into your calculator. You should find that

[tex](5+3)^2 + 32 \div 2 = 8^2 + 16 = 64 + 16 = 80[/tex]

Showing that choice D is the answer.

8.EE.1.2

Which expressions have a value that is between 9 and 10. Select all that apply.

A.) √80

B.) √89

C.) √101

D.) 3 √725

E.) 3 √750

F.) 3 √999

G.) 3 √1010

Answers

Considering that the square root of 9 is of 81 and of 10 is of 100, we have that the expression that has a value between 9 and 10 is given by:

B.) √89

E.) [tex]\sqrt[3]{750}[/tex]

F.) [tex]\sqrt[3]{999}[/tex]

How the square root of a number is used to solve this question?

We suppose numbers n and m, with square roots given, respectively, by n² and m².

Then, the square roots of all numbers between n² and m² have values between n and m.

In this problem, the square roots are given by:

9² = 81.10² = 100.

Hence, the square root of all values between 82 and 99 are values between 9 and 10, thus option B is correct.

As for the cubic root, we apply the same logic and have that:

9³ = 729.10³ = 1000.

Hence options E and F are also correct, as the values between 730 and 999 have cubic roots between 9 and 10.

More can be learned about the value of a expression at https://brainly.com/question/25537936

#SPJ1

suppose the ratio of chocolate cupcakes to vanilla cupcakes baked today how many were chocolate and how many were vanilla 30 POINTS

Answers

Answer:

Ratio Needed

Step-by-step explanation:

In order to solve this problem, we need the ratio. Please give me the ratio, so that I can help you ASAP. Thanks!

Can someone please answer this question. ASAP. put it as an whole number. NOT A FRACTION.

The weight of one serving of chips is 2 1/4 oz. How many servings are there in a 22 1/2 oz bag of chips?

Answers

Answer:

it's 10

Step-by-step explanation:

2 1/4 = 9/4

22 1/2 = 45/2 × 2 = 90/4 and 90/4 ÷9/4 = 10

Answer: 10 servings.

Step-by-step explanation:Divide 22 1/2 by 2 1/4and it woulwould be 10. Of

what is the constant of proportionality, y/x?

Answers

Answer:

7/2

Step-by-step explanation:

y/x   = rise / run  = slope of the line

Pick two convenient integer coordinate points , say 00  and   2,7

   slope =   (7-0) / ( 2-0)  = 7/2

Six years after a tree was planted, its height was 7 feet. Nine years after it was planted, its height was 16
feet. Which of the following equations gives the height y, in feet, of the tree after x years if the tree grows
at a constant rate? Check all of the boxes that apply.
y - 6 = 3(x – 10)
y- 16 = 3(x-9)
y = 3x - 11
y=-3x-23
D
DONE

Answers

Answer:

(b)  y -16 = 3(x -9)(c)  y = 3x -11

Step-by-step explanation:

You are given a couple of points:

  (years, heigh) = (6, 7) and (9, 16)

and asked to write a linear equation that they satisfy. In general, you need to find the slope using the slope formula:

  m = (y2 -y1)/(x2 -x1)

  m = (16 -7)/(9 -6) = 9/3 = 3

__

point-slope equation

The point-slope form of the equation for a line can be used with this information.

  y -k = m(x -h) . . . . . . . . line with slope m through point (h, k)

Using the given points, you can write either of the equations ...

  y -7 = 3(x -6) . . . . . using the first point

  y -16 = 3(x -9) . . . . . using the second point. This matches choice B.

__

slope-intercept equation

Either of the point-slope equations can be rearranged to give the slope-intercept equation.

  y -16 = 3(x -9)

  y = 3x -27 +16 . . . . eliminate parentheses, add 16

  y = 3x -11 . . . . . This matches choice C.

__

Additional comment

The first choice has the right slope, but uses a point not on the line (10, 6).

The last choice has the wrong slope and the wrong y-intercept.

Other Questions
Which environmental characteristic is caused by the Cascades? Why does the purchasing power of money vary based on location? 80 dollars for 16 tickets written as a unit rate, what is it? What type of force might be affecting the motion of these rings? you draw 4 cards from a deck of 52 cards with replacement what are the probabilities Classify3x5 + 5x4 7x3 + 15 bynumber of terms.O polynomial of 5 termsO polynomial of 4 termsO binomialO trinomial Research what you'd need to care for a pet. Select about six items and write their names in the first column of the table. In the second column,write a sentence in Spanish explaining what you would use each item for. Events A and B are independent. Let P(B)=0.4 and P(A and B)=0.13. Find P(A). Write your answer as a decimal rounded to the nearest thousandth. how to rewrite many people read this writer's articles. his articles are usually criticized on politics and migration. with relative pronoun? Solve x2-9x+14=0Would appreciate if anyone could help I need help on my homework DUE TODAY This excerpt best illustrates which feature of a comedy of manners? witty wordplay a commentary on marriage a comparison of country and city life concern with appearance over morality A poll is taken at four different grocery stores to determine how far customers drive to shop at their store. The following information was gathered at the four stores. Customers That Drive < 5 miles Store W 52 of 100 Store X 68 of 75 Store Y 59 of 120 Store Z 28 of 50 If one customer is selected at random from each store, at which store would selecting a customer that drives less than 5 miles be a likely event? What triangle congruence method is used for this triangle? please within 40 min.could anyone help me? Mica is making a paste for an art project. He mixes 8 cups of water with glue. If he wants to make a double batch how many quarts of water does he need?how much water is that in gallons? luke buys shares for 15.26 each they decrease in value by 3.5% work out the new share price A man who heavily smokes has developed lung cancer. The tobacco smoke has caused mutations in some of the cells in his lungs, making them unable to stop reproducing and dividing. He is worried that his children, none of whom smoke, may have inherited the lung cancer from him. Under what circumstances might his concern for his children be justified? If his smoking had already mutated the DNA in the cells in his lungs at the time his children were conceived, the mutations would have been passed to his children. If he inherited a mutation which made him more susceptible to lung cancer, it may have been present in some of the gametes he produced and passed to his children. If the mutation resulted from the duplication of a gene in his lung cells, his children would be at greater risk than if it were caused by a base pair substitution. If the smoke had caused multiple different mutations in his lung cells, it would be more likely that one or more of the mutations would be passed to his children. What are the six thinking hats to stop bad habbits Magnetism: So I need help on this question but please explain how you got the answer so I can learn and better provide evidence :) If you are able to answer this I also am putting a few questions on here for this similar subject for my test and pls explain them all ty!